navle review questions

¡Supera tus tareas y exámenes ahora con Quizwiz!

- pleural effusion DDX includes nothing good: neoplasia, chronic heart failure, hypoalbuminemia, heartworm, diaphragmatic hernia, trauma/hemothorax, lung lobe torsion, etc. Pulmonary edema looks different: think splotchy, cotton-ball lungs where the tissue of the lung itself is heavy with fluid

A 13-year-old male castrated Golden Retriever is presented with respiratory distress. A lateral thoracic radiograph is obtained. (raised trachea, lung lobes floating in the dorsal chest, lung borders outlined clearly by opacity, but the cardiac silhouette is indistinct). A. diaphragmatic hernia B. pleural effusion C. dilated cardiomyopathy D. pulmonary edema E. venal caval syndrome

- this is a very common problem in Thoroughbreds This is a classic description of exercise-induced pulmonary hemorrhage (EIPH) - very common (43-75% of flat racers). Hemorrhage from lungs possibly may occur in ALL Thoroughbreds. Note that only 10% of horses with EIPH may show epistaxis. Look instead for a history of exercise intolerance. On trans-tracheal wash, see alveolar macrophages containing hemosiderin. To maximize your chances of seeing blood in trachea, do endoscopy after exercise. Two other big rule outs for epistaxis are: - ethmoid hematoma - guttural pouch mycosis

A 2 year old Thoroughbred filly has a one-month history of poor race performance. She slows down markedly near the end of races with labored breathing, excessive swallowing, and a bilateral epistaxis which resolves with time. She coughs after exercise, but otherwise acts normal and has a good appetite. With the horse calm and well-rested, no obvious abnormalities are visible on endoscopic exam. A trans-tracheal wash shows alveolar macrophages containing hemosiderin (dark pigment granules left from phagocytized RBCs). What should the owner be told? A. prognosis is poor - suspect gastric ulcer reflex B. the horse should improve on antihistimines C. we need to biopsy for ethmoid hematoma D. this is a very common problem in Thoroughbreds E. this may be a case of bastard strangles

- equine viral arteritis A horse with edema, icterus, and fever should make you think first of equine infectious anemia and equine viral arteritis. Conjunctivitis and rhinitis + ventral edema says equine viral arteritis in this case. Can see limb edema with equine granulocytic anaplasmosis (formerly called equine granulocytic ehrlichiosis) but that choice is not on the list. Respiratory presentation with fever, cough, and eyelid swelling in a horse shipped from Spain or Africa should set off alarm bells in your head for African horse sickness (REPORTABLE!!).

A 4 year old Quarter horse gelding is presented with a 3 day history of poor appetite, and stiffness in the hind limbs. Equine infectious anemia is on the differential diagnostic list. What other diagnosis should be considered for a horse with edema, icterus, and fever? A. African horse sickness B. equine influenza C. equine viral rhinopneumonitis D. equine viral arteritis E. purpura hemorrhagica

- sacroiliac disease Think of sacroiliac disease in an athletic jumping horse with intermittent hindlimb lameness localized in the croup, evidence of back pain, and swelling over the tuber sacrale. Coxofemoral luxation is rare in horses due to a deep acetabulum and the presence of the accessory ligament of the hip, unique to horses Overlapping vertebral spinous processes is a problem of thoracic and lumbar vertebra under the saddle area, seen in short-backed eventing horses Cauda equina can cause gluteal muscle atrophy, but also causes tail paralysis, urinary and fecal incontinence, perineal analgesia or paresthesia, and mild hindlimb ataxia

A 6-year-old hunter jumper horse is presented with a three-month history of poor performance and intermittent shifting hindleg lameness. There is reluctance to bring the hindlimbs forward at a trot and he occasionally drags his hind toes On physical exam, there is poor muscling of the gluteal muscles and some asymmetry to the croup (rump). The horse shows pain and reluctance to ventroflex the back when midline pressure is applied. Which of the following is a top differential diagnosis? A. cauda equina (polyneuritis equi) B. suspensory desmitis C. overlapping vertebral spinous processes D. sacroiliac disease E. coxofemoral luxation

- Northern fowl mite the northern fowl mite remains with its host between blood meals, while the chicken mite feeds only at night and drops off the host during the day Knemidocoptes mutans live on the distal extremities Argas persicus live in poultry houses and intermittently feed on their hosts in tropical and subtropical areas sticktight fleas affect multiple hosts including dogs, cats, horses, and humans. on chickens, they attach to the wattles and skin on the head area

A backyard chicken is presented for laying less eggs than normal and listless behavior. Examination reveals tiny blackish specks moving around the hen's vent Which of the following is the most likely diagnosis? A. sticktight flea B. Knemidocoptes mutans C. chicken mite D. Argas persicus E. Northern fowl mite

- quarantine the horse; take temps on all other horses The horse should be quarantined immediately and all exposed horses monitored closely with twice-daily evaluation of temperature. This presentation is consistent with strangles, a highly contagious respiratory disease. You can confirm the diagnosis with PCR/culture of a nasal swab or swabs of drainage from an abscessed lymph node. Strangles (Strep equi) infects the lymph nodes of the upper respiratory tract and the ensuing lymphadenopathy can "strangle" the horse as seen in this case. It first causes a fever, followed 24-48 hours later by mucopurulent nasal discharge and then lymph node enlargement and abscessation. As the disease progresses, the affected lymph nodes enlarge and eventually rupture and drain purulent exudate. If the retropharyngeal lymph nodes are affected, they can rupture and drain into the guttural pouches, setting up a carrier state. Most cases of strangles respond to supportive care - judicious use of NSAIDs, facilitating abscesses maturation, flushing the abscesses once open/draining. Use of antibiotics depends on severity of clinical signs and the number/ages of horses affected. Antibiotic therapy is controversial - can delay maturation of abscesses and prevent development of protective immunity. Antibiotics are routinely used for horses with metastatic internal abscesses (i.e., bastard strangles), guttural pouch empyema (infection), or those with complications) tracheotomy, severe/prolonged illness).

A couple of days ago, a client brought an 18-month-old filly home from a "kill pen" (was headed for Mexico for slaughter). She just sent you photos of the horse showing thick bilateral mucopurulent nasal discharge and a swollen throat latch. She says that the horse's breathing is stertorous and loud. You are on your way to see the horse, thinking you will need to do an emergency tracheotomy first thing. What are additional preliminary recommendations based on the top differential? A. pass a nasogastric tube to treat esophageal obstruction B. start antimicrobials; limit iodine intake and measure thyroid levels for goiter C. refer for laser ablation treatment for guttural pouch tympany D. will need to biopsy the mass if owner wants to proceed with chemotherapy E. quarantine the horse; take temps on all other horses

- coagulopathy Coagulopathies are common in association with crotalid envenomation. Additional concerns include thrombocytopenia, neurotoxicity, and cardiovascular toxicity. Temporary tracheostomy should be performed immediately in a horse in severe respiratory distress due to upper airway obstruction. The physical examination and further supportive care can be delayed until after an airway is obtained. Additional diagnostics include CBC/blood smear/serum biochemistry/coagulation profile. See hypofibrinogenemia, thrombocytopenia, prolonged prothrombin time, and increased FDPs. Might see echinocytes in heparin or citrate samples, and severity of echinocytosis is associated with severity of envenomation. Rx: IV fluids for hypotension, NSAIDs for analgesia, steroids are controversial, tetanus prophylaxis, antimicrobial prophylaxis, antivenin if owner can afford it/it is available, and occasional blood transfusion.

A horse is presented in severe respiratory distress in southern California after a suspected snake bite. There is significant nasal swelling, flared nostrils, abdominal movement with each breath, stridor, and tachypnea. Aside from local tissue damage and associated upper airway obstruction, what is another major complication of crotalid envenomation? A. chronic visual impairment B. immune suppression C. coagulopathy D. acute hepatic necrosis E. diarrhea

- improve sanitation, install smooth-floored enclosure Bumblefoot (pododermatitis) in guinea pigs usually occurs secondary to poor sanitation, obesity, and wire cage floors or rough bedding. If detected early, switch to smooth-bottom flooring, keep the enclosure clean, and change to a softer bedding. Chlorhexidine soaks and debridement can help the feet. Prognosis is guarded. Avoid penicillins in guinea pigs

A large male guinea pig from a group of four is presented with swollen and scabby hind feet from which Staph aureus is cultured. Which one of the following recommendations is the most appropriate choice? A. cull affected animal, increase ventilation of environment for remaining animals B. isolate affected animal, treat with oral amoxicillin C. improve sanitation, install smooth-floored enclosure D. debridement and topical 1% butenafine cream E. tetracycline-medicated water for all animals

- dermatophytosis This is a typical presentation of ringworm. Up to 15% of clinically normal guinea pigs harbor ringworm. Young animals are more susceptible. Overcrowding, poor sanitation, and other stressors predispose.

A one-year-old female guinea pig is presented with round patches of scaly alopecia on the tip of the nose, ears, periocular areas and forehead. The affected areas do not appear to be pruritic. Skin scrapings from the periphery of lesions and associated broken hairs demonstrate hyphae and arthrospores on direct microscopic examination. Which choice is the most likely diagnosis? A. Cheyletiellosis B. Notoedric mange C. Demodicosis D. Trombiculosis E. Dermataphytosis

- 99% The trick with this kind of question is to pick an imaginary number of animals that you test, like 1000, and fill out your 2x2 table from there. If prevalence is 5%, then there must be 50/1000 cats with FeLV and 950 cats that are disease-free. A 90% sensitive test will correctly call 45/50 positive and incorrectly call 5/50 negative. If 50/1000 animals are infected, then 950/1000 are disease-free. Your 95% specific test will correctly call 902/950 disease-free and incorrectly call 48/950 positive. Now that the boxes are filled, it is easy to calculate PVN: 902/(902+5) = 99%

A practice is using an FeLV test with a sensitivity of 90% and a specificity of 95%. Assuming the prevalence of feline leukemia in the area is 5%, what is the predictive value negative (PVN) of the test? A. 45% B. 88% C. 55% D. 99% E. 75%

- immune-mediated thrombocytopenia The history plus a lab pattern of low platelets, increased bleeding time, and normal APTT and PT suggests immune-mediated thrombocytopenia, the most common cause of spontaneous bleeding in dogs. It can be a primary disorder (idiopathic) or secondary to infection, neoplasia, or certain drug therapies (including vaccines). Twice as common in females; cockers, poodles, and old English sheepdogs are predilected, but it can occur in any breed. Dogs with DIC often present with thrombocytopenia but will also typically have prolonged PT and APTT

A six-year-old female spayed cocker spaniel is presented with a two-day history of lethargy. Upon physical exam, mucosal petechiae and ecchymoses and an ocular hemorrhage in the right eye are noted. Prothrombin time (PT) and activated partial thromboplastin time (APTT) testing are normal. A buccal mucosal bleeding time (BMBT) is performed and is prolonged. Labwork displays high WBCs and neutrophils, low lymphocytes, low eosinophils, and extremely low platelets. Which one of the following diseases is the most likely diagnosis? A. hepatic insufficiency B. immune-mediated thrombocytopenia C. anticoagulant rodenticide toxicity D. von Willebrand's disease E. disseminated intravascular coagulation (DIC)

- strong chance of lifelong FeLV infection 97% of cats positive for FeLV by IFA remain persistently infected and viremic for life. The FeLV ELISA is more sensitive than the IFA (fewer false negatives, so trust a negative test more), so it is a better general screening test with which to start. The FeLV IFA is more specific than the ELISA, (fewer false positives, so trust a positive more) so it is a better confirmatory test for any cats with a positive FeLV ELISA. Three things to remember about FeLV and vaccinations: 1. FeLV ELISA and IFA tests measure antigen, not antibody, so FeLV vaccination does not interfere with testing 2. Vaccinate FeLV-positive cats yearly against respiratory, enteric viruses with inactivated vaccines 3. FeLV vaccination for FeLV-negatives cats has been associated with sarcomas. Vaccinate low on the left hind.

An adopted adult male cat from a shelter with an unknown vaccination history is presented. The cat tests positive for feline leukemia virus (FeLV) infection by IFA. What should the owner be told? A. possible transient FeLV infection B. need an ELISA test in 12 weeks to confirm diagnosis C. strong chance of lifelong FeLV infection D. need a western blot to confirm diagnosis E. cat may be vaccinated for FeLV

- CPV pos that test positive / total CPV positive cats Test sensitivity is the ability of a test to correctly identify those with the disease (true positive rate) whereas specificity is the ability of the test to correctly identify those without the disease (true negative rate) Feline panleukopenia virus is closely related to type 2 canine parvoviruses. CPV-2a and CPV-2b have been shown to cause panleukopenia-like illness in domestic cats

An immunochromatographic test kit for detection of fecal canine parvoviruses antigen is being tested in a local cat shelter where as many as 10% of cats there may have panleukopenia secondary to infection with the canine parvovirus. What is the sensitivity of the test kit? A. CPV pos that test positive / CPV neg that test positive B. CPV neg that test negative / total CPV neg cats C. CPV pos that test positive / total CPV pos cats D. CPV neg that test positive / total cats that test positive E. CPV neg that test negative / total cats that test negative

- ultasonography a skilled ultrasonographer can differentiate between testicular torsion, scrotal hernia, or focal lesions in the testicle this case happened to be a testicular torsion

An intact male whippet presents with an acute onset of swelling and pain in one of his testicles. The right testicle is hemorrhagic and infarcted. Which one of the following tests is helpful to diagnose the cause of the swelling before surgery? A. positive contract cystography B. magnetic resonance imaging C. ultrasonography D. lateral radiography E. nuclear scintigraphy scan

- 3.5-4 years The 4th incisors are actually canine teeth. Cattle age is not always given in a history and estimating age during physical exam helps narrow differentials. References for times of eruption vary by about 6 months because the times of tooth eruption vary among individual animals and breeds. - 1st permanent incisor - 1-1.5 years - 2nd permanent incisor - 2-2.5 years - 3rd permanent incisor - 3.5 years - 4th permanent incisor - 3.5-4 years Full eruption of a new tooth may take another 6 months Remember ruminants don't have upper incisors but instead have a dental pad

At what age do a cow's permanent 4th incisors begin to erupt in the mandible? A. 3.5-4 years B. 7-8 years C. 3 years D. 2-2.5 years E. 5-6 years

- Pasteurella multocida In conjunction with Mycoplasma hyopneumoniae, it causes exudative bronchopneumonia, polyarthritis, and chronic lung lesions. Stressors (parasites, other infections, even weather) can result in severe pneumonia. Best managed by decreasing stressors with improved ventilation and decreased overcrowding. In endemic herds, ABX for sick individuals helps control illness, most likely by preventing secondary bacterial infection (like Pasteurella). Bacterin vaccines give good protection, decrease signs (coughing). Pre-farrowing vaccination of sows decreases colonization in suckling piglets.

Failure to control mild endemic respiratory disease in swine caused by Mycoplasma hyopneumoniae predisposes pigs to complications. Which other organism works in conjunction with Mycoplasma hyopneumoniae to cause exudative bronchopneumonia and polyarthritis? A. Bordetella bronchiseptica B. Hemophilus parasuis C. Fusobacterium necrophorum D. Pasteurella multocida E. Swine influenza virus

- PVP decreases as prevalence decreases As prevalence of a disease goes down, PVP of your serologic test also goes down. That's all you need to know.

For the last 10 years, your state has had a mandatory vaccination program against "pedunculated giblet disease" in fur-bearing turtles and the prevalence of this terrible disease has decreased markedly. How does this decrease in prevalence affect the predictive value positive (PVP) of the best serologic test for pedunculated giblet disease? A. PVP depends on the number tested, not prevalence B. PVP stays the same as prevalence decreases C. PVP increases as prevalence decreases D. PVP decreases as prevalence decreases E. PVP is affected by specificity, not prevalence

- treat affected cows with two classes of dewormer An abomasum with a cobblestone or Moroccan leather appearance in the fall in the southern US is pathognomonic for type II ostertagiasis caused by Ostertagia ostertagi. This finding in a herd with routine deworming suggests anthelmintic resistance. Current recommendations are to deworm with two different classes of dewormer concurrently combined with a refugia program. By deworming only the affected animals (targeted treatment), the other animals serve as a source of refugia.

In September in the Southeastern US, a two-year-old Angus heifer died the previous night. She had a three-week history of poor appetite, unthriftiness, and diarrhea. Several other younger cows in this herd also look unthrifty and have diarrhea. The cattle are vaccinated yearly and dewormed twice a year with pour-on moxidectin. Necropsy reveals that the abomasum is edematous and covered in small umbilicated nodules, 1-2 mm in diameter (cobblestone appearance). What is the most correct action to take? A. treat affected cows with two classes of dewormer B. provide free-choice monensin and thiamine-supplemented feed C. provide loose trace mineral salt to the entire group D. vaccinate against Clostridium perfringens C and D with a multivalent bacterin E. medicate for liver flukes

- 60% of treated horses improve, but less than 25% recover completely The prognosis is guarded to fair for treatment of equine protozoal myeloencephalitis (EPM). Affected animals typically improve by 1-2 ataxia grades (on the Mayhew ataxia scale of 1-5). The earlier treatment is started, the better the outcome. 10-20% of horses relapse within two years of initial diagnosis.

In September, a 15-year old Quarterhorse mare is presented with a five-day history of left head tilt, facial paralysis, depression, and stumbling. The horse is ataxic and knuckles on both forelegs, worse on the left. There is muscle atrophy of the left pectorals and right hindquarters and strips of localized spontaneous sweating over the left trunk. A serum:CSF ratio is consistent with a diagnosis of Sarcocystis neurona. What expectations for recovery should the owner have if she elects to treat the horse? A. prognosis is good if the horse is medicated with doxycycline for 10-12 weeks B. 50% of horses relapse within two years of initial diagnosis C. 60% of treated horses improve, but less than 25% recover completely D. prognosis is grave, regardless of treatment E. 75% of treated horses recover full neurologic function

- none Effective therapy is currently not available for pythiosis. Treatment options often tried include complete surgical excision, local/topical steroids, and systemic antifungals. Without complete surgical excision, the outcome is poor. Occurrence in later summer/early fall, access to standing water, lack of response to antibiotics, "kunkers" and abundant discharge from the lesion, and pruritis are all signs typically seen in infections with pythiosis. Pythium is an aquatic organism, which sporulates in warm water and infects the skin through small skin lesions

It is late summer and a 6-year-old Quarter horse gelding is presented with a mass-like lesion, which started about one month ago. Systemic antibiotics for two weeks did not show any improvement and the mass continued to grow. The horse is pastured with free access to a shallow pond with standing water. At presentation, the large well-circumscribed mass with abnormal tissue resembling excessive granulation tissue is revealed. Within the lesion, draining tracts with abundant discharge and white coral-like structures (kunkers) are found. The area seems to be pruritic. Based on history and clinical presentation, what is the most effective therapy? A. platelet rich plasma B. none C. topical 5-fluoruracil cream D. cryotherapy E. IV sodium iodide

- report outbreak to regulatory authorities the history and necropsy findings are consistent with Newcastle disease, which is reportable. gross lesions such as the petechiae seen here in the proventricular mucosa are not usually seen with low-virulence Newcastle disease NDV occurs worldwide and chickens usually present with acute respiratory disease. Occasionally diarrhea, neurological problems, or depression predominate. sometimes the temperature of the poultry house is increased to decrease morbidity and mortality in flocks with suspected avian influenza Amprolium is a treatment for coccidiosis

Many hens in a large backyard flock recently became ill after the owner purchased four new chickens. The sick hens are gasping, coughing, and sneezing. They have poor appetite and act depressed. Some affected birds have watery green diarrhea and swelling of the head and neck. A few have paralyzed legs and wings, twisted necks, are circling, and have tremors or clonic spasms. Birds are not laying well and some eggs are misshapen with watery albumen. Many of the sickest birds have died. Necropsy of a dead chicken shows petechial hemorrhages on the mucosal surface of the proventriculus and gizzard. What should be done next to address this problem? A. treat all affected chickens with amprolium B. disinfect housing with phenolic compounds, barrier precautions for staff C. report outbreak to regulatory authorities D. increase poultry house temperatures to reduce morbidity E. cull affected birds and vaccinate the remainder

- pour-on eprinomectin Treat this chorioptic mange (also called leg mange) with pour-on eprinomectin or moxidectin (both approved for lactating dairy cattle) Pour-on doramectin and ivermectin as well as eprinomectin and moxidectin are approved for this use in beef cattle Isolation is not practical because of the contagious nature of the disease Assume all animals in the group are infested and treat accordingly Classic chorioptic mange is seen in winter in the northeastern US. Cattle present with crusty, pruritic skin disease around the perineum and caudal upper thigh. On skin scrape, look for long legs and short unsegmented pedicles in chorioptes (in contrast to the short legs and long unsegmented pedicles of sarcoptes)

On a cold day in the Northeastern US, a lactating dairy cow is examined for pruritus and crusts around the perineum. A skin scrape shows some kind of mite with long legs and short unsegmented pedicles. What should be done next? A. quarantine animal B. pour-on eprinomectin C. slaughter D. amitraz SQ E. topical furazolidone

- atrioventricular valves The first sound is caused by closure of the atrioventricular valves (AV valves, mitral and tricuspid) The second sound is the closure of the aortic and pulmonic valves (semilunar valves) A third sound is the end of rapid ventricular filling and a fourth sound is atrial systole (atrial contraction) You can often hear all 4 sounds in horses, but typically hear only S1 and S2 in cattle and small animals

One description of a typical heart sound is "lub-dub". What makes the first heart sound (S1) (i.e., the "lub")? A. atrial contraction B. mitral and semilunar valves C. aortic and pulmonic valves D. ventricular filling E. atrioventricular valves

- vitamin A deficiency Vitamin A deficiency is a problem primarily seen in turtles, but also alligators and other reptiles. Look for swollen eyes, discharge from nose/mouth/eyes and ear infections. It can predispose to a bacterial pneumonia. Inclusion body disease is a viral problem of boa constrictors and some pythons SCUD is a shell disease. Look for pitted shells Herpesvirus causes oral mucosa necrosis, anorexia, regurgitation, oral, ocular discharge, but discharge from the nose and swollen eyes should put vitamin A deficiency at the top of your list. WNV has been diagnosed in farmed alligators with multi-organ necrosis, heterophilic granulomas, heterophilic perivasculitis, and lymphoplasmacytic meningoencephalitis

The parents of a ten-year-old boy present his box turtle with a three-week history of lethargy and anorexia. On physical exam, there is bilateral blepharedema (swollen eyes), as well as ocular and nasal discharge. Which one of the following choices is the most likely diagnosis? A. inclusion body disease B. West Nile Virus C. septicemic cutaneous ulcerative disease (SCUD) D. vitamin A deficiency E. Herpesvirus

- it is an intact male This is the penis of an intact male cat which demonstrates backward-projecting cornified spines on the glans due to the presence of circulating androgens Penile spines are absent in neutered male cats. The presence of spines can help to diagnose cryptorchid cats. Penile spines help toms fertilize queens by breaking through copulatory plugs. This is helpful in a species in which sperm competition inside the female can be fierce

Upon examination, you see backward-projecting cornified spines due to the presence of circulating androgens. What does this finding mean in a cat? A. nasopharyngeal poly, cat will be fine after surgical removal B. lingual poly, can remove, but recurrence is common C. it is an intact male D. cuterebra infestation E. female in estrus

- Salmonella typhimurium S. typhimurium is associated with rectal strictures in growing pigs. It is caused by an ulcerative proctitis that damages rectal tissue. Intestinal spirochetosis is a post-weaning diarrhea seen in the absence of Brachyspira hyodysenteriae (swine dysentery), but similar in presentation to it

What condition is associated with rectal stricture in pigs? A. coccidiosis B. intestinal spirochetosis C. intussusception D. rotavirus E. Salmonella typhimurium

- Clostridioides difficile A history of recent antimicrobial therapy is common in cases of C. difficile associated diarrhea. Adult horses exposed to erythromycin are particularly at risk for C. difficile enterocolitis. C. novyi is the cause of infectious necrotic hepatitis, which is primarily seen in sheep but can also be seen in cattle, hogs, and horses. L. intracellularis is the cause of proliferative enteropathy, resulting in diarrhea and hypoproteinemia in foals and swine. R. equi is a notable cause of pneumonia in older foals characterized by pulmonary abscessation as well as some extrapulmonary manifestations. E. coli can be a cause of septicemia and diarrhea in foals and calves.

When a foal is being treated with erythromycin (for Rhodococcus equi, for example), the mare is at risk for developing enterocolitis due to which one of the following organisms? A. E. coli B. Rhodococcus equi C. Clostridium novyi D. Clostridioides difficile E. Lawsonia intracellularis

- ferrets

Which animal is considered to be as susceptible to acetaminophen toxicity as cats? A. hamsters B. ferrets C. rabbits D. dogs E. rats

- caudal to the transverse processes of T13, L1, and L2 A proximal paravertebral block is performed via placement of local anesthetic just off the midline and caudal to the transverse processes of T13, L1, and L2 vertebrae With a distal paravertebral block, local is placed above and below the ends of the transverse processes of L1, L2, and L4 vertebrae The spinal nerves, T13, L1, and L2 are targeted by a paravertebral block to completely desensitize the flank of a cow Proper placement results in warming of the skin from vasodilation, anesthesia of the skin and body wall, and a curvature of the spine in some cows The latter is caused by relaxation of the epaxial musculature on the affected side; the spine curves towards the opposite side

Which of the following is the correct location for placement of a proximal paravertebral block used to perform a standing laparotomy in a cow? A. caudal to the transverse processes of T13, L1, and L2 B. at the ends of the transverse processes of L1, L2, and L4 C. midway between spine and the ends of the transverse processes of L1, L2, and L3 D. in the spaces between L1, L2, L3, and L4 E. above and below the transverse processes of L1, L2, and L4

- perform a Western blot FIV test The FIV ELISA is the standard screening test to determine if the cat has circulating antibody to FIV. A Western blot antibody test for FIV is the standard confirmatory antibody test. However, the Western blot cannot differentiate antibodies caused by infection from those caused by vaccination. A Western blot is especially important in areas with low FIV prevalence, where the risk of false positive FIV ELISA is higher Cats vaccinated for FIV will test positive for FIV antibodies by FIV ELISA and Western blot Kittens up to 6 months of age born to seropositive queens can also be seropositive, even though they are not infected, due to persistence of maternal antibodies. Seropositive kittens should be retested at greater than 6 months of age. This is different from FeLV testing because the FeLV ELISA and IFA tests measure antigen, not antibody, so FeLV vaccination does not interfere with testing

Which one of the following choices is the most appropriate next step after a 3-year-old stray cat tests positive for feline immunodeficiency virus on a routine FIV ELISA screening test? A. treat with glucocorticoids B. do a Rivalta test to rule out feline coronavirus-related interference C. euthanize D. perform a Western blot FIV test E. recheck serum by FIV complement fixation test

- camelids Birds and reptiles have nucleated, elliptical RBCs.

Which type of animal has ellipsoid blood cells that lack central pallor? A. camelids B. chinchillas C. reptiles D. psittacines E. ferrets

- antigen-antibody complex disease (type III) In type III reactions, antigen-antibody complexes are deposited on the endothelium, stimulating complement and neutrophilic inflammatory response and vascular damage. Look for localized or multi-systemic vasculitis. Classic type III diseases include: - glomerulonephritis - hypersensitivity pneumonitis (think moldy hay) - purpura hemorrhagica (think post-strangles) - anterior uveitis

Which type of immune reaction is occurring in a horse with limb edema secondary to a localized vasculitis? A. cell-mediated immune reactions (type IV) B. immediate hypersensitivity and atopy (type I) C. antigen-antibody complex disease (type III) D. antibody-mediated cytotoxic reactions (type II) E. none of these

- histoplasmosis Think of histoplasmosis in a dog with possible respiratory and diarrheal fungal disease with tiny organisms in macrophages Histo is more a chronic diarrheal and respiratory disease in dogs and a respiratory disease in cats Aspergillus is usually a nasal presentation in dogs Think of cryptococcus in cats with a granulomatous rhinitis and sinusitis

a three-year-old male neutered Labrador retriever from the Ohio river valley basin is presented with chronic large bowel diarrhea and signs of respiratory disease (cough, fever). A lymph node smear shows round to ovoid encapsulated structures inside macrophages and giant cells. The organisms have thin cell walls, with thin clear zone between cell wall and cytoplasm. A. histoplasmosis B. Coccidiodomycosis C. Cryptococcosis D. Blastomycosis E. Asperigillosis

- idiopathic Horner's syndrome is idiopathic in about 50% of dogs and 45% of cats It is caused by a lack of sympathetic innervation to the eye See four ocular signs with Horner's: - miosis (constricted pupil) - protrusion 3rd eyelid (nictitans) - enophthalmos (sunken eye) - ptosis (drooped eyelid) +/- anisocoria Remember: "my third sunken toe" (miosis, 3rd lid protrudes, sunken eye, ptosis) Look for classic increased anisocoria in the dark. Other diagnostic tests depend on where suspect primary disease

What is the most common cause of Horner's syndrome in dogs? A. otitis media B. neck bite wounds C. brachial plexus avulsion D. retrobulbar neoplasia E. idiopathic

- multiple myeloma MM is a malignancy of plasma cell origin. Malignant plasma cells arise from bone marrow and produce immunoglobulins, resulting in hyperglobulinemia. Hyperglobulinemia causes many of the clinical signs seen with MM. Clinical signs may include lethargy, weight loss, bone pain or fractures, CNS signs, capillary bleeding, visual disturbances, organomegaly, and heart murmurs. Diagnosis requires two or more of the following: - monoclonal gammopathy on serum electrophoresis - Bence Jones proteinuria - lytic bone lesions on radiographs - plasma cell infiltration of the bone marrow

A 10 year old male neutered West Highland white terrier is presented with a recent history of PU/PD, lethargy, and weight loss. Exam reveals a grade II heart murmur and mild muscle wasting along the lumbar spine. Significant lab abnormalities include pancytopenia, hyperglobulinemia, hypercalcemia, and isosthenuria. Bone marrow cytology reveals sheets of plasma cells. What diagnosis choice is most likely, based on these findings? A. multiple myeloma B. osteosarcoma C. aplastic anemia D. systemic lupus erythematosis E. Babesia canis

- buy only vaccinated chicks This chicken is likely to have ocular manifestations of Marek disease. There is no treatment, but the problem is preventable when chicks are vaccinated at hatching or before two weeks of age. Diffuse infiltration of mononuclear cells can turn the irises of affected chickens from a normal yellow color to a pale tan to gray color ("gray eye"). Cellular infiltration can also occur in many other tissues (skin, nerves, viscera), and there may be palpable tumors in the muscle and skin. Marek disease is common in commercial poultry. Diagnosis is usually based on enlarged nerves and lymphoid tumors in the viscera. Lymphoid leukosis is a key differential. Absence of bursal tumors helps distinguish Marek disease from leukosis. But if you find bursal tumors, mark disease is still on the ddx list pending other tests, like immunochemistry. Chicks as young as three weeks can show Marek disease, but lymphoid leukosis is typically seen in chickens more than 14 weeks old.

A 12-week-old chicken from a young backyard flock is presented for evaluation because of weight loss, decreased appetite, and diarrhea. The irises in the affected bird are lighter than normal with irregular pupillary margins, and the hen does not have a normal pupillary light reflex. Several other young birds appear to be growing poorly compared to the rest of the flock. Based on the presumptive diagnosis, how can this problem be prevented going forward? A. isolate unaffected animals; breed from survivors of disease B. use only mycoplasma-resistant bloodlines for new additions to flock C. buy only vaccinated chicks D. cull birds from the same genetic line E. collect pharyngeal swabs from affected birds; antibiotic rx for entire flock based on culture and sensitivity

- gastric volvulus Even if the radiograph looks somewhat normal, the history should strongly suggest GDV. GDV of more than 180 degrees can be difficult to diagnose, especially if the stomach is not distended. In a 270 or 360 degree volvulus, the rest of the stomach can appear to be in a relatively normal position

A 12-year-old Labrador Retriever is presented for abdominal distension. An orogastric tube was passed and gas and gastric contents were removed from the stomach. Radiographs were obtained after gastric decompression. On this study, the stomach is mildly gas-filled. The fundus appears to be in normal position, however, the duodenum is visible cranial to the stomach on the VD view. The spleen is not visible on the VD view: however, on the lateral view it appears mildly enlarged. The colon, cecum, and small bowel loops are mildly displaced on the right side and caudally. Which one of the following choices is the most likely diagnosis? A. gastric volvulus B. normal abdominal radiographs C. hepatomegaly D. mesenteric torsion E. splenic torsion

- pleural effusion DDX includes nothing good: neoplasia, chronic heart failure, hypoalbuminemia (secondary to protein-losing nephropathy/liver disease), heartworm, diaphragmatic hernia, trauma/hemothorax, lung long torsion to name a few. pulmonary edema looks different - think splotchy, cotton-ball lungs, where the tissue of the lung itself is heavy with fluid

A 13-year-old male castrated Golden Retriever is presented with respiratory distress. A lateral thoracic radiograph is obtained. You note the raised trachea, with lung lobes floating in the dorsal chest; lung borders are outlined clearly by opacity, but the cardiac silhouette is indistinct. What is the primary abnormality visible in the radiograph? A. vena caval syndrome B. pleural effusion C. dilated cardiomyopathy D. diaphragmatic hernia E. pulmonary edema

- high ringbone high ringbone - periostitis and osteoarthritis of the proximal interphalangeal joint leading to exostoses very common in horses - can be due to chronic wear and tear, overuse, or secondary to a traumatic episode pedal osteitis is demineralization of the solar margin of the distal phalanx, usually due to inflammation bone spavin is osteoarthritis of the distal intertarsal, tarsometatarsal, and less commonly, the proximal intertarsal joints

A 15 year old quarter horse mare is presented for right front lameness grade 3/5 (lameness consistently visible at the trot). She becomes sound after an abaxial nerve block. What is the top differential diagnosis? A. pedal osteitis B. high ringbone C. exostosis of the second metacarpal D. spavin E. chronic proliferative synovitis

- sacroiliac disease Think of sacroiliac disease in an athletic jumping horse with: - intermittent hind limb lameness localized in the croup (rump) - evidence of back pain - swelling over the tuber sacrale ("hunter's bumps") Horses with hunter's bumps can be sound if the injury has completely healed, but lameness can be observed if the injury is recent or has never resolved. It is important to rule out other causes of hind end lameness or back pain/strain. Ultrasound per rectum or local anesthesia infiltration of the SI region can help confirm the diagnosis. However, most often treatment is administered instead (inject area with steroids and analgesics) and the response to therapy monitored. Coxofemoral luxation is rare in horses due to a deep acetabulum and the presence of the accessory ligament of the hip, unique to horses. Overlapping vertebral spinous process ("kissing spines") is a problem of thoracic and lumbar vertebra under the saddle area, seen in short-backed eventing horses (hunter/jumpers, dressage). Cauda equina can cause gluteal muscle atrophy, but also causes tail paralysis, urinary and fecal incontinence, perineal analgesia or paresthesia, and mild hind limb ataxia.

A 6 year old hunter-jumper horse is presented with a 3 month history of poor performance and intermittent shifting hindleg lameness. There is reluctance to bring the hind limbs forward at a trot and he occasionally drags his hind toes. On physical exam, there is poor muscling of the gluteal muscles and some asymmetry to the croup (rump). The horse shows pain and a reluctance to ventroflex the back when midline pressure is applied. Which one of the following is a top differential diagnosis? A. cauda equine (polyneuritis equi) B. sacroiliac disease C. suspensory desmitis D. coxofemoral luxation E. overlapping vertebral spinous processes

- cauda equina syndrome Look for pain in the lumbosacral area (elicited by tail raise, hindlimb extension), LMN hindlimbs, especially sciatic nerve damage at L7-S1 (lack withdrawal), +/- urinary/fecal incontinence, +/- self-mutilation of tail, perineum, or pelvic limb. Lesion due to compression of cauda equina at L7-S1 (lumbosacral stenosis) Can be congenital (abnormal development dorsal arch L7-S1, small dogs, Border Collies) or acquired (degenerative changes, big dogs, especially German Shepherd, Rottweiler Boxer) Wobbler syndrome (cervical vertebral instability) is a congenital cervical spinal cord disease. Affected dogs are born with vertebral canal stenosis which can worsen later in life due to intervertebral disc disease, vertebral malformation and/or ligamentous hypertrophy. Usually seen at C5-C6 or C6-C7 in Dobies > 5 years, Great Danes < 2 years. Diskospondylitis is a good second choice on your ddx of a large middle aged dog presenting with lumbosacral pain Due to bacterial/fungal infection of intervertebral disk and adjacent vertebral bodies Look for systemic signs like fever, weight loss, anorexia.

A 7 year old male German Shepherd is presented with a history of weakness in the hind limbs, urinary incontinence and recent obsessive chewing around his tail area. Dorsiflexion of the tail over the back and extension of the hind limbs elicits a painful response. He does not withdraw each hind leg when a toe is pinched, but bears weight on the hindlimbs. Patellar reflexes are normal. What is the clinical diagnosis? A. radiculoneuritis B. cauda equina syndrome C. hip dysplasia D. diskospondylitis E. Wobbler syndrome

- mesenteric volvulus nearly always a fatal condition, mesenteric volvulus is not common. found more frequently in german shepherds than other breeds presentation resembles gastric torsion volvulus - rapid onset of shock, abdominal pain and distension, retching and vomiting a linear foreign body would result in a plication of the intestines

A 9 year old German Shepherd is presented for non-productive retching. Physical exam reveals severe abdominal pain and in shock. After starting fluid therapy, a lateral abdominal radiograph is made. You note marked gas distension of the bowel. The dog is taken to surgery and laparotomy reveals sections of the bowel that are diffusely darker in color. Which one of the following choices is the most likely diagnosis? A. gastrointestinal ulceration B. intussusception C. mesenteric volvulus D. linear foreign body E. intestinal lymphosarcoma

- DIC A lab pattern of low platelets, increased bleeding time and across the board increases in aPTT, PT, TT, and FDP suggests DIC. It is a complex hemostatic defect characterized by enhanced coagulation and fibrinolysis, secondary to other diseases. Fibrinolysis and depletion of clotting factors leads to hemorrhage. Many diseases can precipitate DIC. This case presentation suggests hemangiosarcoma Remember your "H diseases" associated with DIC: heartworm, heart failure, hemolytic anemia, hemangiosarcoma, hemorrhagic gastroenteritis, hepatic disease (especially hepatic lipidosis in cats) GDV, mammary gland carcinoma and pancreatitis can also lead to DIC

A 9 year old German shepherd is presented with unchecked bleeding from a cut on the gums above the right canine tooth. The owner relates that the dog has lost weight and had an episode of collapse three days ago, but he recovered. On physical exam, the gums are pale with petechiae and ecchymotic hemorrhages. There is tachycardia and a palpable cranial abdominal mass. A coagulation profile shows the following: - low thrombocytes - increased buccal mucosal bleeding time - increased activated partial thromboplastin time, prothrombin time, and thrombin time - increased fibrin degradation products What disorder of coagulation best fits this pattern? A. hepatic insufficiency B. disseminated intravascular coagulation C. anticoagulant rodenticide toxicity D. idiopathic thrombocytopenia E. von Willebrand's disease

- thyroid function, BUN/creatinine This presentation of vision loss in older cats suggests retinal detachment, typically secondary to hypertension. Chronic renal disease may lead to hypertension and retinal detachment. Exam, history, BUN/creatinine and urine specific gravity will rule this disease in or out. Hyperthyroidism can also cause hypertension. Your exam and history, plus a measure of T4 will help you evaluate this condition. Other diseases associated with retinal detachment include your alphabet diseases and 2 T's: FIP, FeLV, FIV, toxoplasmosis, and trauma

A 9 year old male neutered DSH cat is presented with a 4-week history of occasional disorientation and bumping into things. The cat does not visually track a ball rolled in front of him and has decreased pupillary light reflexes in both eyes. On fundoscopic exam, the retinal vessels and disc are obscured by something in the way. What testing is indicated to assess the two most common underlying causes of this condition in cats? A. CSF analysis, CBC B. adrenal function, arterial pressure C. thyroid function, BUN/creatinine D. urinalysis, pancreatic function, fasting blood glucose E. hepatic function, serum globulins

- biopsy mass, submit for histopathology This is a malignant melanoma in a cat, but without histopathology, there is no way to know conclusively what the mass is. Destruction of maxillary incisors on the upper right side is suggestive of a malignant pathology. Dark pigmented color can indicate melanoma, but these tumors can also be non-pigmented. In cats, squamous cell carcinoma is the most common malignant oral tumor. Most oral malignant melanomas are highly invasive and readily metastasize. The prognosis is guarded to poor. Surgical resection in the rostral areas of the mouth may be curative or at least extend survival. Local recurrence is common. Oral melanomas and ameloblastomas are more common in dogs than in cats. In dogs, subungual (nail bed) melanomas are locally invasive but have a lower rate of metastasis (30-60%); most cutaneous melanomas in dogs are benign.

A 9-year-old male castrated cat is presented with a two- to three-week history of anorexia, halitosis, and ptyalism. A smooth, black oral mass is evident on the upper gingiva and hard palate that seems to have invaded and replaced the upper right incisors. Based on these findings, what is the best step to take next? A. initiate masitinib mesylate trial, monitor for response B. abdominal ultrasound C. debulk, immunosuppressive-dose prednisolone D. biopsy mass, submit for histopathology E. thoracic radiographs, radiation therapy

- Northern fowl mite The Northern fowl mite remains with its host between blood meals, while the chicken mite feeds only at night and drops off the host during the day. Therefore, you are likely to find the northern fowl mite on the chicken during the day in the vent area, whereas the chicken mite is found in the chicken house during the day. Knemidocoptes mutans are also called scaly-leg mites and live on the distal extremities. They cause the legs to become thick and crusty. Argas persicus are soft ticks that live in poultry houses and intermittently feed on their hosts in tropical and subtropical areas. Sticktight fleas affect multiple hosts, including dogs, cats, horses, and humans. When on chickens, they attack to the wattles and skin on the head area.

A backyard chicken is presented for laying less eggs than normal and listless behavior. Examination reveals tiny blackish brown specks moving around the hen's vent. Which one of the following is the most likely diagnosis? A. Sticktight flea B. Northern fowl mite C. Knemidocoptes mutans D. Argas persicus E. Chicken mite

- serial progesterone testing This is the most commonly used hormonal assay method for estimation of the approximate ovulation day in bitches. Progesterone levels are typically measured every two to three days once significant cornification is seen in the vaginal cytology of a proestrus bitch. Baseline progesterone is 0-1 ng/ml. About 48 hours prior to ovulation, a rise in progesterone (greater than 2 ng/ml) is seen, corresponding with the luteinizing hormone surge, which is the actual stimulus for ovulation. The progesterone level increases to 4-10 ng/ml on ovulation day. For natural breeding or insemination with fresh or chilled semen, dogs are usually bred at least twice, typically on days 2 and 4 after the estimated ovulation date. Serial daily measurement of serum LH is also a recognized method of estimation of ovulation day in bitches and is in fact the most definitive diagnostic test available. However, LH testing is hampered by expense, necessity for daily testing, and problems with availability of tests. Administration of exogenous LH has not been shown to be safe and effective in inducing estrus in dogs. While vaginoscopy is a useful adjunct in breeding management, progesterone measurement is a more accurate method. Estrogen measurement is of little value for estimation of ovulation day because peak levels vary from dog to dog and do not necessarily correlate with the fertile period. Ovarian ultrasonography is not commonly used in clinical practice in dogs.

A female dog is approaching estrus and the owner has consulted you about planning a breeding for this dog. She is aware of the signs of proestrus and plans to contact you when those are apparent in her dog. In addition to vaginal cytology, which one of the following will be the most useful to you in planning the timing of breeding? A. determination of serum estrogen levels B. timed administration of luteinizing hormone C. serial progesterone testing D. vaginoscopy E. ovarian ultrasonography

- granulomatous meningoencephalitis The lesion in this case localizes to the left cerebellum causing the left hypermetria and a right paradoxical head tilt. Granulomatous meningoencephalitis often affects this area.

A four-year-old Rat Terrier is presented with a three day history of progressive stumbling and falling. Physical exam reveals a right head tilt, left sided hypermetria, generalized ataxia and vertical nystagmus. Which one of the following choices is at the top of the differential list? A. amyotrophic lateralizing sclerosis B. granulomatous meningoencephalitis C. ascending and descending myelomalacia D. fibrocartilagenous embolism E. central pontine myelinolysis

- antibodies against p2-myelin protein The top differential for this horse with cauda equina signs PLUS cranial nerve signs is polyneuritis equi. Horses with this condition can have circulating antibodies against P2-myelin protein, but many false positives. Other diseases to rule out in this case would be equine herpes virus 1 and equine protozoal encephalomyelitis Unfortunately, there is no definitive Rx and prognosis is poor for functional recovery

A four-year-old stallion is presented with urine scald, fecal retention, tailhead rubbing, and a right head tilt. Rectal examination reveals atonia of the anus and rectum, however, no fracture is palpated. Which one of the following tests help provide evidence of the top differential diagnosis? A. search the pasture for yellow star thistle B. antibodies against P2-myelin protein C. tibial muscle and nerve biopsies D. cervicothoracic spinal radiography E. immunoblot against DNA fragments from P. tenuis

- supplement dietary zinc In pigs, zinc deficiency causes parakeratosis. Zinc supplementation will resolve clinical signs. Parakeratosis may resemble exudative epidermitis (greasy pig disease) caused by Staph. hyicus - more seen in younger, suckling piglets and treated with antibiotics. Sarcoptic mange is typically pruritic and treated with antiparasitics like ivermectin or pigs may be sprayed with lindane or malathion

A group of 2 1/2-month-old feeder pigs are presented with non-pruritic keratinized skin lesions and mild lethargy. One severely affected animal is depressed and anorexic. What treatment is most appropriate for the presumptive diagnosis? A. Copper sulfate bath or sprays B. High dose trimethoprim-sulfonamide 7-10 days C. Supplement dietary zinc D. Spray with malathion (0.05%) E. Ivermectin SQ now, repeat in 2 weeks

- necropsy affected calves; submit lung for Mycoplasma bovis testing This is a classic outbreak scenario of M. bovis (respiratory signs, swollen joints and head tilt/suppurating ear involvement). It is best to confirm the diagnosis by culture and/or PCR from necropsy samples of affected lung tissue. Speciation is necessary as other less virulent mycoplasmas can also cause pneumonia. M. bovis has ways of evading the host immune response, which leads to persistent infections and poor response to treatment with antimicrobials. A significant proportion of affected animals remain chronically ill and unthrifty.

A group of six-month-old Holstein heifers is experiencing an outbreak of respiratory disease and lameness. Examination shows swollen hot joints and one calf has suppurative drainage from one ear and a head tilt. Aspirated joint fluid from an affected calf is purulent. Auscultation suggests consolidation of ventral lung fields. Rectal temperatures range from 103.5-105 (normal is 101.5-102.5) What is the most appropriate action to take next? A. treat chronic cases with parenteral antimicrobials; add organic iodine to feed B. quarantine the entire herd; call state veterinarian C. divide sick animals into treatment groups; perform on-farm antibiotic efficacy trial D. necropsy affected calves; submit lung for Mycoplasma bovis testing E. culture silage samples for listeriosis; vaccinate healthy animals with autogenous bacterin

- treat the entire group with an anthelmintic Those not showing clinical signs may still be subclinically infected and benefit from treatment. Products used to treat lungworms also impact GI nematodes, so treating an entire group will impact GI nematode refugia and anthelmintic resistance. This should be factored into your treatment decisions. Lungworm infection is mostly seen in yearlings on pasture in the summer and falls. The primary clinical sign is coughing. 4-Ipomeanol toxicity from moldy sweet potatoes or Perilla mint can present similarly but necropsy shows pulmonary edema and emphysema.

A group of yearling stocker cattle on summer pasture are presented for coughing in approximately 30% of the group. The owner has treated the coughing animals with antibiotics with no response. When the herd is gathered for examination, the cough worsens and some animals become dyspnic. Some of the cattle look thin and have rough hair coats. Necropsy of one animal shows lungworms (Dictyocaulus viviparous). What is the appropriate action for this herd? A. call the state vets office to report a possible foreign animal disease B. do nothing, as this is a self-limiting problem C. check for moldy sweet potatoes and Perilla mint D. treat the coughing animals with an anthelmintic E. treat the entire group with an anthelmintic

- metabolic alkalosis Think of hypochloremic metabolic alkalosis due to hydrochloric acid sequestration in the abomasum of a cow with a displaced abomasum Abomasal hypomotility, ongoing HCl secretion into the abomasum, and partial abomasal outflow obstruction all contribute. Metabolic alkalosis can also be due to HCl loss in monogastric animals who vomit In a similar way, metabolic acidosis can be due to HCO3 loss in saliva if animal cannot swallow, or from diarrhea

A high-producing dairy cow that freshened three weeks ago is off-feed. On physical exam her heart and respiratory rates are within normal limits. Rumen motility is decreased and a urine test for ketone bodies is positive. There is no evidence of mastitis and the uterus is clear of infection. On the left side, a high-pitched musical "ping" is audible via stethoscope during percussion over the ribs on a line between the elbow and tuber coxae (hip). What acid-base abnormality is most likely in this cow? A. respiratory alkalosis B. metabolic alkalosis C. depends on severity of displacement D. respiratory acidosis E. metabolic acidosis

- root canal therapy The fracture has left the pulp exposed, requiring endodontic treatment. Extraction is not necessary if the radiographs and periodontal tissue are normal.

A lion is presented with a tooth fracture of an upper left premolar, exposing the pulp. Dental radiographs are normal and no periodontal disease is present. Which one of the following choices is the best treatment? A. restore the tooth with light cured resin B. wait until the exposure fills in, then re-radiograph C. root canal therapy D. do nothing E. extraction

- dermatophytosis This is a typical presentation of ringworm (Trichophyton mentagrophytes). As with most species, ringworm in immunocompetent animals is usually self-limiting with good husbandry and sanitation. Up to 15% of clinically normal guinea pigs harbor T. mentagrophytes. Young animals are more susceptible. Overcrowding, poor sanitation and other stressors predispose. Treatment will speed resolution of infection and minimize chances of infecting other animals or people, but it is somewhat labor intensive and long term (4-8 weeks). Systemic: Itraconazole or terbinafine for 4-8 weeks. Topical: Enilconazole or miconazole shampoo once or twice a week. Decontaminating the environment every 14 days is also important for control. Can use enilconazole or concentrated chlorine laundry blead solutions.

A one-year-old female guinea pig is presented with round patches of scaly alopecia on the tip of the nose, ears, periocular areas and forehead. The affected areas do not appear to be pruritic. Skin scrapings from the periphery of lesions and associated broken hairs demonstrate hyphae and arthrospores on direct microscopic examination. Which choice is the most likely diagnosis? A. dermatophytosis B. trombiculosis C. cheyletiellosis D. demodicosis E. notoedric mange

- lavender foal syndrome This presentation is consistent with lavender foal syndrome, an inherited lethal neurologic disorder that affects Arabian foals with a dilute coat color. It's a recessive genetic disorder and newborn foals present as in this case. Affected foals are typically euthanized within days of birth. There is no treatment. A genetic test is available and affected animals should not be bred. Botulism in foals less than 1 month old is characterized by progressive symmetric motor paralysis with muscular tremors. Affected foals cannot stand for more than a few minutes at a time. Overo lethal white syndrome is characterized by colic and meconium impaction due to ileocolonic agangliosis. It occurs in Paint foals with blue eyes that are the result of breeding two Overos (a type of coat pattern in Paint horses). Myasthenia gravis is not reported in horses. Neurologic signs seen with hyperammonemia associated with portosystemic vascular anomalies are not seen until affected foals are 2-3 months old and eating more grain and forage.

A pale-colored Egyptian Arabian foal is presented for evaluation within a few hours of a dystocia with assisted vaginal delivery. The foal has opisthotonos, nystagmus, limb rigidity and paddling, and has not yet stood and nursed. What is the presumptive diagnosis based on the information available? A. botulism B. lavender foal syndrome C. myasthenia gravis D. portosystemic vascular anomaly E. overo lethal white condition

- require original owner consent or court order to release information According to the AVMA's Principles of Veterinary Medical Ethics: "Ethically, the information within veterinary medical records is considered privileged and confidential. It must not be released except by court order or consent of the owner of the patient. Veterinarians should secure a written release to document that request."

A person calls to say she adopted a friend's dog and requests a copy of the dog's medical record and radiographs. The former owner is a client at your clinic. Which one of the following choices is the most appropriate action to take? A. give a summarized record with personal identifiers (vet, owner name) blocked out B. give caller a copy of medical record and radiographs C. after physical exam, start a new medical record documenting previous issues D. require original owner consent or court order to release information E. give caller a copy of medical record only

- treponematosis, hutch burn Treponematosis (rabbit syphillis, vent disease) is a venereal disease of rabbits caused by Treponema paraluis cuniculi. It affects the genitalia, and may affect the eyes and nose. Hutch burn is caused by wet and dirty floors, affecting the anus and genitalia. Remember that cauda equina neuritis (polyneuritis equi) in horses may present with urine scald on the thighs. Other signs include a weak tail, hypotonic anus, urine dribbling and fecal retention. There may be a history of rubbing or chewing the tail head.

A rabbit is presented with inflamed and chapped membranes of the anus and genital region. The genital area is scalded and raw, with brownish crusts and purulent exudate. What two conditions top the differential diagnosis list? A. pasteurellosis, ulcerative pododermatitis B. tularemia, cystitis C. glomerulonephritis, coccidiosis D. treponematosis, hutch burn E. myxomatosis, moist dermatitis

- inguinal hernia Inguinal hernia is a COMMON problem in pigs Intersex syndrome is in both pigs and goats but is extremely rare Perineal hernia is more of a problem in middle-aged pure bred dogs Cryptorchidism is a testicle retained, not protruding

A seven-week-old pot-bellied pig is presented with a posterior swelling. Which of the following is the most likely clinical diagnosis? A. testicular torsion B. inguinal hernia C. intersex syndrome D. cryptorchidism E. perineal hernia

- cervicothoracic: C6-T2 weak, hypotonic (lower motor neuron) forelimbs and spastic paresis (upper motor neuron) hindlimbs are signs of cervicothoracic lesion may see worse signs in fores than hinds note: this is the opposite presentation as Schiff-Sherrington syndrome (severe acute spinal cord trauma in the region of T3-L3), where in lateral recumbency, the thoracic limbs are rigid and extended and the pelvic limbs appear flaccid in comparison can localize T3-L3 lesions by checking cutaneous trunci reflex - the lesion is usually 1-2 vertebrae cranial to where the reflex disappears with C1-C5, would expect UMN signs in all four limbs, usually worse in hinds with T3-L3, would expect UMN hind limb signs and normal forelimbs

A stray dog is presented after being hit by a car. The dog has hypotonic forelimbs and spastic paresis in the hinds. All four limbs have proprioceptive deficits and sensation loss - signs are worse in the forelimbs Where is the lesion? A. cranial cervical: C1-C5 B. thoracolumbar: T3-L3 C. cervicothoracic: C6-T2 D. cannot say without cutaneous trunci reflex results E. lumbosacral: L4-S3

- cervicothoracic C6-T2 Weak, hypotonic (lower motor neuron) forelimbs and spastic paresis (upper motor neuron) hindlimbs are signs of a cervicothoracic lesion. May see worse signs in fores than hinds. Note: This is the opposite presentation as Schiff-Sherrington syndrome - severe acute spinal cord trauma in the region of T3-L3, where in lateral recumbency, the thoracic limbs are rigid and extended and the pelvic limbs appear flaccid in comparison. However, pelvic limb reflexes are normal to increased, as would be expected with an upper motor neuron lesion. Can localize T3-L3 lesions by checking cutaneous trunci reflex - the lesion is usually 1-2 vertebrae cranial to where the reflex disappears. With C1-C5, would expect UMN signs in all four limbs, usually worse in hinds. With a T3-L3, would see UMN hind limb signs and normal forelimbs.

A stray dog is presented after being hit by a car. The dog has hypotonic forelimbs and spastic paresis in the hinds. All four limbs have proprioceptive deficits and sensation loss-signs are worse in the forelimbs. Where is the lesion? A. thoracolumbar T3-L3 B. cannot say without cutaneous trunci reflex results C. lumbosacral L4-S3 D. cervicothoracic C6-T2 E. cranial cervical C1-C5

- pancreatitis Pancreatitis is most likely based on the feline pancreatic lipase immunoreactivity assay, hypothermia, anorexia, and bloodwork. Clinical signs of feline pancreatitis are generally vague, and most often include lethargy and anorexia. Icterus occurs in some cats, usually when cholangiohepatitis is also present. Vomiting occurs only in 33% of feline pancreatitis case-patients, and abdominal pain in only 25%. In contrast, 90% of dogs with pancreatitis present with vomiting and anorexia, and 50% have abdominal pain

A ten-year-old neutered male cat is presented with vague signs of anorexia, occasional vomiting, and lethargy. On physical exam, the patient is somewhat dehydrated. He has a 99.1 degree fever. Bloodwork: - ALB low - ALT high - BUN high - CL low - feline pancreatic lipase immunoreactivity high - ALK PHOS high - AMYL high - K low - WBC high Which of the following diseases is most likely, based on these findings? A. hepatic lipidosis B. pancreatitis C. feline infectious peritonitis D. cholelithiasis E. toxoplasma gondii

- choanal atresia Choanal atresia is one of the most common congenital problems of South American camelids. Choanal atresia occurs when the caudal nares (choanae) fail to open during embryologic development. Can be unilateral or bilateral and may cause complete or partial blockage.

A three-day-old female alpaca (cria) is presented in respiratory distress. The cria's cheeks flare noticeably during inspiration, and the distress is more pronounced during nursing, when the animal gasps and inhales milk. What is the most likely diagnosis? A. lung lobe torsion B. choanal atresia C. diaphragmatic hernia D. mitral stenosis E. wry face

- treat with potassium permanganate Saprolegnia is a saprophytic fungus-like pathogen that commonly causes disease in freshwater fish. Treat fish with potassium permanganate, hydrogen peroxide, or formalin once causative factors have been addressed for this Saprolegnia infection. Low water temperatures are a major predisposing factor for infection. Other possible causes include poor sanitation and the presence of decaying organic material (or dead fish)

A trout fishery reports that many of the fish have gray-white, puffy growths on their fins, gills, and eyes. What action should you take to address the top differential? A. depopulation is the only appropriate measure B. supplement the fish with vitamin C C. administer praziquantel to all the fish D. ask if there is any zinc in the housing E. treat with potassium permanganate

- Conium maculatum (poison hemlock) Coniine is found in seeds and the mature plants; g-coniceine is found in young growing plants Poison hemlock is toxic to all livestock and humans. Signs of toxicity develop one to two hours after ingestion and are usually fatal Signs include nervousness, trembling, weakness especially of the hind limbs, weak pulse, irregular heart rate, recumbency, coma, and death A mousy odor exuding from the urine and breath is pathognomonic Ingestion of poison hemlock during gestation causes arthrogryposis and other congenital defects in cattle, goats, and pigs

A two-year-old Angus cow is presented with weakness of the hind limbs and a staggering gait. On physical exam, the heart rate and respiratory rate are slow and irregular, and the pupils are dilated. The cow's breath and urine smell like the odor of mouse urine. Suddenly the cow's pulse becomes rapid and thready, she collapses, and dies of apparent respiratory failure. Which one of the following plants is most likely to have caused this spectrum of clinical signs? A. Veratrum spp. (False hellebore, skunk cabbage) B. Tetradymia spp. (Horsebrush) C. Conium maculatum (Poison hemlock) D. Pinus ponderosa (Western yellow pine) E. Centaurea spp. (Knapweed, Yellow star thistle)

- this is a very common problem in Thoroughbreds This is a classic description of exercise-induced pulmonary hemorrhage - very common. Hemorrhage from lungs possibly may occur in all Thoroughbreds. Note that only 10% of horses with EIPH may show epistaxis - look instead for a history of exercise intolerance. On trans-tracheal wash, you'd see alveolar macrophages containing hemosiderin (dark pigment granules left from phagocytized RBCs). To maximize your chances of seeing blood in trachea, do endoscopy after exercise. Two other big rule outs for epistaxis are ethmoid hematoma and guttural pouch mycosis.

A two-year-old Thoroughbred filly has a one-month history of poor race performance. She slows down markedly near the end of races with labored breathing, excessive swallowing, and a bilateral epistaxis which resolves with time. She coughs after exercise, but otherwise acts normal and has a good appetite. With the horse calm and well rested, no obvious abnormalities are visible on endoscopic exam. What should the owner be told? A. prognosis is poor - suspect gastric ulcer reflux B. this may be a case of bastard strangles C. the horse should improve on antihistamines D. this is a very common problem in Thoroughbreds E. we need to biopsy for ethmoid hematoma

- H1 histamine blocker; corticosteroid Before surgical excision, pre-operative treatments to shrink the tumor and minimize degranulation is key with H1 histamine blockers and corticosteroids. In addition, proton pump inhibitors and/or H2 histamine blockers can also be used to prevent or treat histamine-associated gastric ulceration. Further pre-surgical workup is also important. Elements include: CBC, blood chemistry panel, urinalysis, and regional lymph node aspiration. CBC changes with mast cell tumors can include eosinophilia, basophilia, and regenerative or non-regenerative anemia. Boston terriers are predisposed to developing mast cell tumors. Other predisposed breeds include: pug, Shar-pei, Staffordshire bull terrier, golden and Labrador retrievers. Affected animals can be pruritic and some may have underlying allergies.

An 8-year-old female spayed Boston terrier is presented with a solitary well-circumscribed mass about 2 cm in diameter on the left hip area. The owner reports that the mass occasionally appears inflamed, and that the dog has a history of allergies. A fine needle aspirate of the mass is performed and the specimen stained using Diff-Quik. There is a uniform population of round cells with basophilic granules in the aspirate, which is consistent with a well-differentiated mast cell tumor. Based on the clinical presentation, cytological findings and presumptive diagnosis in this patient, the owner opts for surgical resection. What pre-operative treatment, if any, is indicated before surgery? A. adrenergic agonist (epinephrine); bronchodilator (aminophylline, theophylline) B. nonsteroidal anti-inflammatory analgesics; H2 histamine blocker C. bacteriocidal beta-lactam antibiotics; calcium channel blocker (vasodilator) D. H1 histamine blocker (diphenhydramine); corticosteroids E. no pre-operative treatment is indicated

- skin scraping and microscopic exam Like most other animals, South American camelids get sarcoptic mange (scabies). A skin scrape and microscopic exam can rapidly confirm the diagnosis. Treatment is an avermectin-family drug, SQ every 2 weeks. One report suggests amitraz is also effective. If you see a severely pruritic dog that looks like this, think of bovine sarcoptic mange. If you see a severely pruritic cow that looks like this, think of bovine sarcoptic mange. In contrast, some things may look like mange, but aren't.

An alpaca is presented in poor body condition with an unkempt coat, patchy alopecia and severe pruritus and crusting of the extremities. What is the most appropriate diagnostic step? A. test skin crusts for contagious ecthyma parapoxviruses B. check thyroid status and bluetongue titers C. tell Yoda you found his brother D. punch biopsy E. skin scraping and microscopic exam

- hypothyroidism Tail alopecia and tragic facial expression are classic signs of hypothyroidism in dogs. Other areas of bilateral alopecia that are common in hypothyroidism are the ventral and lateral trunk, caudal thighs, ventral neck and dorsum of the nose. Hyperadrenocorticism commonly has truncal alopecia and also presents with obesity. It is more common in toy breed dogs.

An obese 8 year old Labrador Retriever is presented for annual vaccinations. The owners claim that they have restricted his diet and try to exercise him, but he still keeps the weight on. There is alopecia on the dorsum of his tail and he has an unusual worried facial expression. Which of the following choices is the most likely diagnosis? A. hypoadrenocorticism B. hypothyroidism C. sarcoptic mange D. male pattern alopecia E. hyperadrenocorticism

- edema disease Edema disease is caused by shiga toxin-producing E. coli Look for severe acute illness ranging from peracute death with no signs to CNS involvement with ataxia, paralysis, and recumbency in healthiest pigs 1-2 weeks after weaning Hemagglutinating encephalomyelitis virus is almost exclusive to piglets less than 4 weeks old. Two clinical presentations: vomiting and wasting disease and encephalitic Clostridium perfringens type C enteritis, also called enterotoxemia in other animals, is characterized by a hemorrhagic diarrhea in 1-3 day old piglets Porcine proliferative enteritis is principally a diarrheal disease of growing finishing (40-80 lb) pigs and young breeding pigs Epidemic transmissible gastroenteritis in non-immune pig herds characterized by high morbidity and high mortality in piglets less than 1 week old

An outbreak of diarrheal disease of piglets has occurred which affected the healthiest animals in the herd, one to two weeks after weaning. Some affected piglets had no signs except peracute death. Other affected piglets exhibit diarrhea, ataxia, paralysis, and recumbency. What condition is at the top of the differential diagnosis list? A. Clostridium perfringens type C enteritis B. Edema disease C. Porcine proliferative enteritis D. Epidemic transmissible gastroenteritis E. Hemagglutinating encephalomyelitis virus

- Spirocerca lupi A disease of dogs in the Southern US and tropical climates, Spirocerca lupi (esophageal worms) make reactive granulomas of variable size in the esophageal, gastric, or aortic walls. Spirocercosis may also lead to aneurysm in the thoracic aorta or an ossifying spondylitis of the posterior thoracic vertebrae. Typically asymptomatic, but large granulomas can cause esophageal obstruction. Large granulomas may become neoplastic (osteosarcoma, fibrosarcoma). Some dogs develop spondylitis or enlargement of the extremities characteristic of hypertrophic osteopathy. All of the other choices are gastric parasites: - Ollulanus tricuspis is an uncommon gastric parasite of cats - Physaloptera spp. is a stomach worm that may cause vomiting, anorexia, dark feces in dogs and cats - Haemonchus spp., Ostertagia spp. and Trichostrongylus spp. are found in the abomasum of ruminants - Gasterophilus spp. are the larvae of horse bot flies, found in the stomach of horses

During the necropsy of an eight-year-old mixed breed dog from the Southern US, reactive granulomas in the esophagus containing bright red worms, 40 mm to 70 mm long are found. Which choice is the most likely diagnosis? A. Ollulanus tricuspis B. Gastrophilus spp. C. Haemonchus placei D. Spirocerca lupi E. Physaloptera spp.

- atrioventricular valves The first heart sound is caused by closure of the atrioventricular valves (mitral and tricupsid). The second sound (S2) is the closure of the aortic and pulmonic valves (semilunar valves). The third sound (S3) is the end of rapid ventricular filling and a fourth sound (S4) is atrial systole (atrial contraction). You can often hear all 4 sounds in horses, but typically hear only S1 and S2 in cattle and small animals.

One description of a typical heart sound is "lub-dub". What makes the first heart sound (S1) (the lub)? A. mitral and semilunar valves B. atrial contraction C. aortic and pulmonic valves D. atrioventricular valves E. ventricular filling

- classical swine fever AKA hog cholera - the best tissues to submit are tonsils, maxillary or submandibular lymph nodes, mesenteric lymph nodes, spleen, ileum, and kidney Glasser's disease is an acute bacterial infection, characterized by different combinations of meningoencephalitis, polyserositis, and polyarthritis and can contribute to bacterial pneumonia

Over the past two weeks, several pigs in a herd have been febrile and depressed. Many were constipated, then had diarrhea. A few were uncoordinated and one had seizures. A few have died. Necropsy revealed petechial hemorrhages on the kidneys and larynx, and a hemorrhagic urinary bladder. Of the following choices, which one is most consistent with a presumptive diagnosis? A. Streptococcus suis infection B. Swine dysentery C. Glasser's disease D. Erysipelas E. Classical swine fever

- Type II - antibody binds cell antigen and activates complement Antibody binds antigen on a cell, then the antibody-antigen complex activates complement, causing cell lysis In general, think of rare, autoimmune skin diseases characterized by varying presentations of ulceration, crusting, pustules, and vesicles FYI: delayed hypersensitivity is the same as a type IV reaction

Pemphigus foliaceus, pemphigus vulgaris, and bullous pemphigoid are examples of which type of immune-mediated disease? A. Type III - antigen-antibody complexes deposited in tissues B. Type II - antibody binds to cell antigen and activates complement C. Type IV - antigen triggers cell-mediated cytokine release, activates macrophages and production of cytotoxic T cell D. Type I - immediate IgE-mediated hypersensitivity E. delayed hypersensitivity

- vaccinate all animals with Clostridium novyi toxoid Active immunization with Clostridium novyi toxoid before the late summer is the most effective way to control and prevent infectious necrotic hepatitis (black disease) in sheep. Because vaccinated sheep have long-term immunity after only one shot, only new introductions to the flock (lambs and sheep brought in from other areas) need to be vaccinated. Reducing the number of snails (intermediate hosts for fluke cercaria) with molluscicides or by fencing off wet areas may not be practical due to expense or amount of pasture lost. Likewise, use of flukicides is complicated by the need for careful timing and long withdrawal times for meat and milk.

Several young sheep in a large herd have died suddenly with a diagnosis of infectious necrotic hepatitis secondary to fluke infestation. Which choice is the most practical and effective control measure you can take next to prevent future cases? A. treat ponds with copper sulfate molluscicide against lymnaeid snails B. Clostridium hemolyticum bacterin for animals under 3 years C. Clorsulon antihelmintic for whole herd D. vaccinate all animals with Clostridium novyi toxoid E. high-dose penicillin for clinical cases

- chorioptic mange Chorioptes infestation is often seen as a component of pastern dermatitis, a multi-factorial disease that is very common in draft horses, especially those breeds with heavy feathering. It is also called scratches, greasy heel, and dermatitis verrucosa.

This is the most common mite in horses. It is characterized by long legs and short pedicles. Most commonly it is found on the hind legs where it causes pruritis. A. chorioptic mange B. trombiculiasis C. demodecosis D. sarcoptic dermatitis E. cheyletiellosis

- IV fluids; NSAIDs, frequent milk feedings Fever, diarrhea, and sudden death in eight-week-old calves is highly suggestive of septicemia due to salmonellosis. Isolate sick calves. Use of antibiotics is controversial as they may prolong recovery and shedding and yield a carrier calf. However, if an animal is septic, it needs parenteral antibiotics. Prognosis is poor with neonatal salmonella and deaths can approach 100% in affected calves. In adults, antibiotics may yield a clinical cure, but Salmonellae can establish in the biliary system and intermittently shed into the GI system, leading to environmental contamination. Prevention is dependent on which species of Salmonella is causing the problems: host-adapted or environmental

Two eight-week old calves are presented down and extremely weak. They are depressed and lying in pools of foul-smelling brown diarrhea with a small amount of blood. Another calf died suddenly the previous night with no prior signs. The down calves are dehydrated, with rectal temperatures of 105.2 and 105.6 (normal is 101.5-103.5). What is the treatment plan? A. corticosteroids; amprolium in water; rumensin in feed B. cull sick calves; prophylactic oxytetracycline in food for well animals C. isolate sick calves; oral electrolytes D. IV fluids; NSAIDs; frequent milk feedings E. immunize calves and adult cattle with MLV vaccine; probiotics for sick calves

- rare disease; early diagnosis improves prognosis You need a very sensitive test if: - disease is rare - early Dx improves prognosis - the disease is highly lethal or consequences of missing a case are severe Remember that a highly sensitive test will have very few false negatives. That means if a test is highly sensitive, you can trust a negative test.

Under what conditions is a very sensitive test used? A. treatment does not affect prognosis; non-infectious diseases B. lethal disease; highly prevalent disease C. zoonoses; untreatable diseases D. rare disease; early diagnosis improves prognosis E. common disease; infectious diseases

- idiopathic Horner's syndrome is idiopathic in about 50% of dogs and 45% in cats. It is caused by a lack of sympathetic innervation to the eye. See four ocular signs with Horner's: - miosis (constricted pupil) - protrusion 3rd eyelid (nictitans) - enophthalmos (sunken eye) - ptosis (drooped eyelid) +/- anisocoria Remember "My 3rd Sunken Toe" (miosis, 3rd lid protrudes, sunken eye, ptosis). Look for classic increased anisocoria in the dark. Other diagnostic tests depend on where suspect primary disease. If you can get a primary diagnosis, you must treat that. Examples include guttural pouch mycosis in horses, brachial plexus avulsion, otitis media, etc.

What is the most common cause of Horner's syndrome in dogs? A. retrobulbar neoplasia B. otitis media C. neck bite wounds D. brachial plexus avulsion E. idiopathic

- interruption of inhibitory neuron input from lumbar spinal cord With severe trauma between T2-T13, inhibitory pathways are interrupted and cervical intumescence neurons (C6-T2) are released, causing extensor hypertonia in the forelimbs. Think of Schiff-Sherrington syndrome when you see a combination of forelimb extensor rigidity and hind limb flaccid paralysis, in an animal that has just had major spinal trauma, like being hit by a car.

What is the neuroanatomic cause of extensor rigidity in the forelimbs of a cat with Schiff-Sherrington syndrome? A. brachial plexus trauma B. polyradiculoneuritis of peripheral nerve sheaths at cervical intumescence C. interruption of inhibitory neuron input from lumbar spinal cord D. subdural white matter compression at C6-T3 E. damage to thoracic spinocerebellar tracts in superficial white matter

- interruption of inhibitory neuron input from lumbar spinal cord with severe trauma between T2-T13, inhibitory pathways are interrupted and cervical intumescence neurons (C6-T2) are "released", causing extensor hypertonia in the forelimbs think of Schiff-Sherrington syndrome when you see a combination of forelimb extensor rigidity and hind limb flaccid paralysis, in an animal that has just had major spinal trauma, like being hit by a car

What is the neuroanatomic cause of extensor rigidity in the forelimbs of a cat with Schiff-Sherrington syndrome? A. polyradiculoneuritis of peripheral nerve sheaths at cervical intumescence B. brachial plexus trauma C. damage to thoracic spinocerebellar tracts in superficial white matter D. subdural white matter compression at C6-T3 E. interruption of inhibitory neuron input from lumbar spinal cord

- create transfaunation donors The rumen is permanently fistulated in the left paralumbar fossa to allow cows to be donors for transfaunation of rumen contents to other cows. Transfaunation is used to treat other cows with conditions like ruminal acidosis, vagal indigestion, and peritonitis. Fresh ruminal fluid contains microbes (bacteria and protozoa), volatile fatty acids, microbial proteins, vitamins, minerals, and other buffers. Ruminal fluid can also be collected via siphon from a stomach tube or from animals at the slaughter house. Ruminal fistulas are also used to study ruminant physiology.

What is the purpose of placing a permanent rumen fistula in some cattle? A. treat traumatic reticuloperitonitis in the fistulated animal B. allow for direct administration of medications into rumen long-term C. manage vagal indigestion in the fistulated animal D. this procedure is not done on a permanent basis in cows E. create transfaunation donors

- Anaplasma The causative organism of equine granulocytic anaplasmosis was originally classified as Ehrlichia equi, but is now called Anaplasma phagocytophilum due to DNA sequencing studies. Do not confuse equine anaplasmosis, a necrotizing vasculitis, with bovine anaplasmosis, which primarily causes an anemia with icterus and fever. Another name change occurred with the causative organism of Potomac horse fever, from Ehrlichia risticii to Neorickettsia risticii Potomac horse fever presents as a febrile colitis/diarrhea, with laminitis 3-5 days after diarrhea in horses of all ages: a big rule out is salmonella (think septicemia/fever + diarrhea)

What kind of organism causes equine granulocytic anaplasmosis? A. Ehrlichia B. Protozoa C. Anaplasma D. Spirochete E. Chlamydia

cystine and urate "I can't C U"

What types of stones can you NOT see on radiographs? (Dr. Lawson said this was good to know for NAVLE)

- Clostridioides difficile A history of recent antimicrobial therapy is common in cases of C. difficile associated diarrhea Adult horses exposed to erythromycin are particularly at risk for C. difficile enterocolitis Clostridium novyi is the cause of infectious necrotic hepatitis, which is primarily seen in sheep but can also be seen in cattle, hogs, and horses Lawsonia intracellularis is the cause of proliferative enteropathy, resulting in diarrhea and hypoproteinemia in foals and swine Rhodococcus equi is a notable cause of pneumonia in older foals characterized by pulmonary abscessation as well as some extrapulmonary manifestations E. coli can be a cause of septicemia and diarrhea in foals and calves

When a foal is being treated with erythromycin (for Rhodococcus equi, for example), the mare is at risk of developing enterocolitis due to which one of the following organisms? A. E. coli B. Clostridioides difficile C. Clostridium novyi D. Lawsonia intracellularis E. Rhodococcus equi

- ferrets Like cats, ferrets metabolize acetaminophen slowly because they are deficient in glucuronyl transferase. There are unsubstantiated web reports of pot bellied pigs reacting poorly to acetaminophen, but we have not yet seen per-reviewed papers to verify this. There are recommended acetaminophen dosages for dogs, rabbits, rodents, and guinea pigs, but high doses are toxic to dogs and other animals, so don't overdo it. In cats, toxicity can occur with 10-40 mg/kg. There is a case report of a kitten with fatal toxicity after being given an empty acetaminophen bottle to play with. In one report, 3 of 4 dogs showed clinical signs of methemoglobinemia at 200 mg/kg, but toxicity can be seen at lower dosages with repeated exposures.

Which animal is considered to be as susceptible to acetaminophen toxicity as cats? A. rabbits B. rats C. hamsters D. dogs E. ferrets

- old, small-breed dogs; Cavalier King Charles Degenerative valve disease is the most common cardiac disease of dogs, accounting for about 75% of all canine cardiovascular disease Roughly 60% of affected dogs have myxomatous degeneration of the mitral valve. Older, small-breed dogs have a higher incidence and Cavalier King Charles Spaniels are prone Don't confuse endocardiosis with infective endocarditis, which is due to bacterial infection of the cardiac valves and endocardial muscle

Which group is most at risk of developing endocardiosis (degenerative valve disease)? A. cattle without reticular magnets; Charolais B. old, small-breed dogs; Cavalier King Charles Spaniel C. sheep or goats pastured above 2000 meters; Saanens D. obese cats, any breed; Burmese cats E. young horses ingesting Perilla mint; Standardbreds

- palmar digital This block would anesthetize at least the palmar third of the foot, including the navicular bone. It desensitizes 70-80% of the foot in most horses All of the other nerve blocks can desensitize the navicular bone too, but would not be as specific as a palmar digital nerve block The abaxial sesamoidean nerve block would anesthetize the entire foot and much of the pastern A low four-point nerve block would desensitize the fetlock and areas distal to it The median and ulnar nerve block would anesthetize the carpus and areas distal to it The tibial and peroneal nerve block would anesthetize the tarsus and areas distal to it

Which nerve block would most specifically relieve lameness resulting from fracture of the navicular bone? A. median and ulnar B. palmar digital C. abaxial sesamoidean D. tibial and peroneal E. low four-point

- T13, L1, and L2 The paravertebral nerve block targets these nerves. It can be performed via two techniques: the proximal or distal PV block. The proximal block places local anesthetic in the space just caudal to the transverse processes of the vertebrae - T13, L1, and L2. The distal block is placed at the ends of the transverse processes of the vertebrae - L1, L2 and L4 as the nerves gradually course caudally after they exit the spinal foramen. Proper placement of the anesthetic results in warming of the skin from vasodilation, anesthesia of the skin and body wall, and the curvature of the spine in some cows. The latter is caused by relaxation of the epaxial musculature on the affected side; the spine curves in a convex manner.

Which of the following nerves are targeted with a paravertebral block used to perform a standing laparotomy in a cow? A. L1, L2, and L4 B. L1-L3 C. L1-3 and S1-5 D. T13, L1, and L2 E. L2, L3, S1 and S2

- T13, L1, and L2 The spinal nerves, T13, L1, and L2 must be blocked to completely desensitize the flank of a cow The paravertebral nerve block targets these nerves. It can be performed via two techniques: the proximal or distal PV block The distal block is placed at the ends of the transverse processes of the vertebrae - L1, L2, and L4 as the nerves gradually course caudally after they exit the spinal foramen Proper placement of the anesthetic results in warming of the skin from vasodilation, anesthesia of the skin and body wall, and a curvature of the spine in some cows The latter is caused by relaxation of the epaxial musculature on the affected side; the spine curves in a convex manner

Which of the following nerves are targeted with a paravertebral block used to perform a standing laparotomy in a cow? A. T13, L1, and L2 B. L2, L3, S1, and S2 C. L1, L2, and L4 D. L1-3, and S1-5 E. L1-3

- liver and muscle Damage to liver and muscle cells of dogs and cats causes increased serum levels of alanine aminotransferase. ALT is considered a leakage enzyme. High levels are normally found in the cytoplasm of healthy cells. When hepatic or muscle cells are damaged, ALT leaks into adjacent tissue where it is picked up by the venous circulation. Other animals such as horses, ruminants, pigs, and birds do not have high levels of ALT inside cells. Aspartate aminotransferase (AST) is the predominant leakage enzyme in these animals.

Which of the following pairs of tissues can both cause increased serum alanine aminotransferase (ALT) levels in dogs and cats? A. spleen and adrenals B. heart and kidneys C. liver and muscle D. kidneys and pancreas E. pancreas and intestine

- liver and muscle damage to liver and muscle cells of dogs and cats causes increased serum levels of ALT ALT is considered a leakage enzyme. High levels are normally found in the cytoplasm of healthy cells other animals such as horses, ruminants, pigs, and birds do not have high levels of ALT inside cells - AST is the predominant leakage enzyme in these animals (aspartate aminotransferase)

Which of the following pairs of tissues can both cause increased serum alanine aminotransferase (ALT) levels in dogs and cats? A. spleen and adrenals B. heart and kidneys C. pancreas and intestine D. liver and muscle E. kidneys and pancreas

- inability to close the mouth Idiopathic trigeminal neuritis is characterized by acute onset of flaccid jaw paralysis Idiopathic facial nerve paralysis results in the inability to move the eyelid, lip, or ear and dryness of the eyes and mouth Masticatory myositis is characterized by pain on opening the mouth and swelling of the muscles of mastication (acute) or atrophy of the temporalis and masseter muscles with the inability to open the mouth due to fibrosis (chronic) Dysphagia, dysphonia, and stridor are most often associated with dysfunction of the vagus nerve Circling and head tilt toward the side of the lesion with no other signs is a common presentation of vestibulocochlear nerve Concurrent facial nerve paralysis and Horner's syndrome may be present with middle- and inner-ear infections

Which one of the following choices includes the cardinal sign of trigeminal neuritis? A. paralyzed eyelid, ear or lip on one or both sides of the face B. circling and head tilt toward side of lesion, no other signs C. masseter muscle pain associated with chewing D. inability to close the mouth E. dysphagia, dysphonia and stridor

- proton pump inhibitor Omeprazole is a proton pump inhibitor that decreases gastric acid secretion. Omeprazole inhibits the H/K-ATPase proton pump at the luminal surface of gastric parietal cells. Parietal cells normally secrete hydrogen ions into the stomach, a key component of acidic HCl. Other drugs that decrease gastric acid secretion include the H2-receptor antagonists cimetidine, ranitidine, and famotidine and a synthetic prostaglandin E1 analog called misoprostol. Carprofen, etodolac, deracoxib, meloxicam, and firocoxib are all nonsteroidal anti-inflammatory drugs (NSAIDs) that may cause gastric acid secretion.

Which one of the following choices is the mechanism of action of omeprazole? A. H2-receptor antagonist B. synthetic prostaglandin E1 analog C. cyclooxygenase blocker D. proton pump inhibitor E. beta-adrenergic receptor agonist

- dystocia, overfeeding in dry period, Ca-P imbalance in feed Cows are also predisposed to metritis by: - retained placenta - contaminated calving environment - abortion - malnutrition In cattle, the causative bacterial organisms isolated most often are Trueperella pyogenes alone or with Fusobacterium necrophorum or other gram-negative anaerobes. Specific diseases associated with bovine metritis or endometritis include brucellosis, leptospirosis, trichomoniasis, and bovine campylobacter.

Which one of the following sets of conditions predispose a cow to metritis? A. Strep. agalactiae mastitis, retained placenta, laminitis B. milk fever, malnutrition, excess dietary zinc C. dystocia, overfeeding in dry period, Ca-P imbalance in feed D. agalactia, milk fever, bovine vibriosis E. contaminated calving environment, abortion, hypomagnesemia

- dystocia, overfeeding in dry period, Ca-P imbalance in feed cows are also predisposed to metritis by retained placenta, contaminated calving environment, abortion, and malnutrition in cattle, the causative bacterial organisms isolated most often are Trueperella pyogenes alone or with Fusobacterium necrophorum or other gram-negative anaerobes

Which one of the following sets of conditions predispose a cow to metritis? A. contaminated calving environment, abortion, hypomagnesemia B. agalactia, milk fever, bovine vibriosis C. milk fever, malnutrition, excess dietary zinc D. Strep agalactiae mastitis, retained placenta, laminitis E. dystocia, overfeeding in dry period, Ca-P imbalance in feed

- antigen-antibody complex disease (type III) In type III reactions, antigen-antibody complexes are deposited on the endothelium, stimulating complement and a neutrophilic inflammatory response and vascular damage classic type III diseases include: glomerulonephritis, hypersensitivity pneumonitis, purpura hemorrhagica, and anterior uveitis

Which type of immune reaction is occurring in a horse with limb edema secondary to a localized vasculitis? A. antibody-mediated cytotoxic reactions (type II) B. none of these C. cell-mediated immune reactions (type IV) D. antigen-antibody complex disease (type III) E. immediate hypersensitivity and atopy (type I)

- nothing Extreme accentuation of sinus arrhythmia (bradycardia), markedly slow during expiration, is a normal finding in brachycephalic breeds If the dog is otherwise healthy and shows no signs of illness, there is nothing to treat

While conducting a routine physical on a four-year-old male intact bulldog, an irregular heart rhythm with a slow rate that is markedly slower on expiration is audible during auscultation. What should be done next? Temp = 102.1 (normal is 99.5-102.5) HR = 65 (normal is 60-120) RR = 24 (normal is 15-34) A. refer for cardiology consult B. chest radiograph, CBC, blood chemistry panel C. nothing D. blood pressure assessment E. echocardiogram

- infectious bursal disease caused by birnavirus, shed in feces and transferred barn to barn via fomites the above findings are most common in older birds. in young birds you also subclinically see immunosuppression via destruction of immature lymphocytes in bursa, thymus, and spleen

what is the presumptive diagnosis in a chicken with the following necropsy finding? (enlarged, balloon-like hemorrhagic bursa of Fabricius, which is the site of hematopoiesis in birds) A. Marek's disease B. Histomoniasis C. renal dystrophy D. infectious bursal disease E. avian spirochetosis

- create transfaunation donors transfaunation is used to treat other cows with conditions like ruminal acidosis, vagal indigestion, and peritonitis

what is the purpose of placing a permanent rumen fistula in some cattle? A. treat traumatic reticulperitonitis in the fistulated animal B. allow for direct administration of medications into rumen long-term C. this procedure is not done on a permanent basis in cows D. create transfaunation donors (transfer of microorganisms from a healthy animal to a sick animal) E. manage vagal indigestion in the fistulated animal


Conjuntos de estudio relacionados

NCLEX practice questions documentation

View Set

Henry David Thoreau - "Civil Disobedience"

View Set